For the function f(x)=|x-3 | select all the true statements. A. The function is increasing on the interval [3, 5) B. The function is increasing of the Interval (0, 3) C. The vertex is (0, 3) D. The vertex is (3, 0). E. The y-intercept is (0, 3). F. The y-intercept is (3, 0).​

Answers

Answer 1

Answer:

D and E

----------------------

See attached graph to help with answer choices.

Given function:

f(x) = | x - 3 |

Answer choices:

A. The function is increasing on the interval [3, 5) - False, correct interval is [3, + ∞)B. The function is increasing on the Interval (0, 3) - False, as shown aboveC. The vertex is (0, 3) - False, the vertex is (3, 0)D. The vertex is (3, 0) - TRUEE. The y-intercept is (0, 3) - TRUEF. The y-intercept is (3, 0) - False, the correct one is given above
For The Function F(x)=|x-3 | Select All The True Statements. A. The Function Is Increasing On The Interval

Related Questions

The volume of a rectangular prism is 5,890.625cm3 . if the height is 14.5 cm and the length is 25 cm, with is the value of the width ?
A. 16.25 cm B. 16.5 cm C. 16.75cm D. 16.97cm

Answers

Answer:

Step-by-step explanation:

The answer is A. 16.25cm.

Explanation:-

We know,

The Volume of Prism  =height*width*length;

Here, In the question Height and length is given.

So We have to do a simple calculation to find,width=

(Vol of Prism)/Heigth*length;

=> 5890.625/(14.5*25)

=>16.25 cm .

We don't need to worry about dimensions because all are in the same cm dimensions.

Read More :

brainly.com//prism

Answer:

A. 16.25 cm

Step-by-step explanation:

To solve for the volume of a rectangular prism, we would use the formula V = l·w·h, where:

"l" is the base length

"w" is the base width

"h" is the height of the prism

In this problem, we are given the volume, height, and length of the rectangular prism. That means we need to find the value of w, the base width. To do this, we plug our known values into the equation and solve for w.

5,890.625 = (14.5)(25)w

To get w by itself, we first divide each side of the equation by 14.5. Remember that when you divide a number by itself, they cancel out.

(5,890.625)/(14.5) = (14.5)(25)w/(14.5)

406.25 = (25)w

Next, we can divide each side of the equation by 25 to get our answer.

406.25/25 = (25)w/25

w = 16.25 cm

The type-1 error (false positive) for a carbon monoxide detector installed in your house is 0.05 and its type-2
error (false negative) is 0.03. The probability that a gas heater malfunctions and releases carbon monoxide is
very low, only 0.000007.

What is the probability that the carbon monoxide detector will not go off?

O 0.9998642
O 0.9499936
O 0.0500064
O 0.0001358

Answers

The probability that the carbon monoxide detector will not go off is approximately 0.9998642 (rounded to 7 decimal places),

Option A is the correct answer.

We have,

The probability of the carbon monoxide detector not going off can occur in two ways: either there is no carbon monoxide present, or there is carbon monoxide present but the detector fails to detect it.

The probability of the detector failing to detect carbon monoxide when it is present (type-2 error) is 0.03, and the probability of the gas heater malfunctioning and releasing carbon monoxide is 0.000007.

So the probability of the detector failing to detect carbon monoxide when it is present is:

0.03 x 0.000007

= 0.00000021

The probability of there being no carbon monoxide present is 1 minus the probability that the gas heater malfunctions and releases carbon monoxide, which is:

1 - 0.000007

= 0.999993

Now,

So the overall probability of the detector not going off is the sum of the probabilities of these two events:

0.999993 + 0.00000021

= 0.99999321

Therefore,

The probability that the carbon monoxide detector will not go off is approximately 0.9998642 (rounded to 7 decimal places), which is option A.

Learn mroe about probability here:

https://brainly.com/question/14099682

#SPJ1

In IXL I need help im bad :(

Answers

Answer:

Wilma will pay $23.44 total.

Step-by-step explanation:

Cost of sweater = (+) $24.49

Discount = (-) $5

Shipping cost = (+) $3.95

Solve:

24.49 - 5 = 19.49

19.49 + 3.95 = 23.44

Wilma will pay $23.44 total.

The Ferris Wheel can carry 4 passengers in each of its 15 cars in one ride. How many passengers can it carry on a total of 35 rides?

Answers

The Ferris Wheel can carry 2100 passengers on a total of 35 rides.

According to the question,

Number of cars in the Ferry Wheel = 15

Number of passengers in each car = 4

∴ Number of passengers in 15 cars = 15×4 = 60

So, the number of passengers on the Ferry Wheel = 60

Now,

The number of passengers the Ferry Wheel carries in one ride = 60

∴ The number of passengers in 35 rides = 60×35 = 2100.

Hence the Ferris Wheel can carry 2100 passengers on a total of 35 rides.

For more word problems on multiplication,

https://brainly.com/question/2076545

https://brainly.com/question/29079711

Triangle AABC, right angled at C, is given. Height and the median from point C form an angle y.
The measure of larger acute angle of AABC is:
A 45°-
B
C
D
60° +
90°
24
92
2
92
4

Answers

The measure of the larger acute angle of ΔABC is: α = 45° + φ/2. Option A.

How do you solve for  the larger acute angle of ΔABC ?

Let's denote the angles of triangle ΔABC as follows:

∠A = x

∠B = y

∠C = 90° (right-angled triangle)

Let D be the midpoint of AB, so CD is the median. Let E be the point on AB such that CE is the height from point C.

Since CD is the median, we know that angle ∠ECD = φ.

In right-angled triangle ΔCEB, we have:

∠CEB = 90° - y

Now, let's examine triangle ΔCED. We know that the sum of the angles in a triangle is 180°. Therefore:

∠CED + ∠CEB + ∠ECD = 180°

Substitute the known values:

∠CED + (90° - β) + φ = 180°

Since ∠CED and ∠A are supplementary angles, we can also write:

∠CED = 180° - x

Now substitute this value into the previous equation:

(180° - x) + (90° - y) + φ = 180°

Simplify the equation:

270° - x - y + φ = 180°

Subtract 90° from both sides:

180° - x - y + φ = 90°

From this equation, we get:

x + y = 90°

Substitute this value back into the equation involving φ:

180° - (90°) + φ = 90°

Simplify:

90° + φ = 90°

Therefore, the measure of the larger acute angle of ΔABC is:

x = 45° + φ/2 (option a)

the above answer is in response to the full question below;

Triangle ΔABC, right angled at C, is given. Height and the median from point C form an angle φ. The measure of larger acute angle of Δ ABC is:

a. 45⁰ + φ/2

b. 60⁰ + φ/2

c. 90⁰ - φ/2

d. 2φ

find more exercise on finding acute angle of ΔABC;

https://brainly.com/question/2515183

#SPJ1

will mark brainliest

Answers

Answer:

$74.66

Step-by-step explanation:

Plot the points on the graphing calculator (x = length of HDMI cord, y = cost). Then generate a linear regression model:

y = 3.68x + 1.06 (approximately)

x = 20, so y = 74.66

$74.66 is the correct answer.

Making an Informed Decision Using the Quantitative Reasoning Process
Step 1: Understand the Problem
Keep the Old Car or Buy a Used Car
Manny is an online student who currently owns an older car that is fully paid for. He drives, on average, 190 miles per week to commute to work. With gas prices currently at $
2.9 per gallon, he is considering buying a more fuel-efficient car, and wants to know if it would be a good financial decision.
The old car Manny owns currently gets 18 miles per gallon for average fuel efficiency. It has been a great vehicle, but with its age, it needs repairs and maintenance that average $
770 per year (as long as nothing serious goes wrong).
The newer, more fuel-efficient car that he is looking at to purchase will cost a total of $
6,500 over a three-year loan process. This car gets 32 miles per gallon and would only require an average of $
10 per month for general maintenance. To help make a decision, Manny wants to calculate the total cost for each scenario over three years. He decides to use the quantitative reasoning process to do this.
Step 2: Identify variables and assumptions
Manny has identified some key variables for his situation. Select the two variables that should be removed from his list because they do not apply to the current situation.
Manny’s graduation date from elementary school.
The cost of repairs on the old car.
The cost of gas.
The number of miles Manny drives each week.
Manny’s wages.
The loan cost of the used car.
The year Manny was born.
The cost of the used car.
Manny has also made a list of key assumptions he will be using to make his decision. Check the boxes for the three assumptions from his list that are not useful in this scenario.
Manny assumes that nothing will go seriously wrong with either car during this three year period.
Manny assumes the loan period will be 3 years.
Manny assumes gas costs will remain about the same over the three years.
Manny assumes he will continue to go running in the mornings every weekday for the next three years.
Manny assumes there are 52 weeks in a year.
Manny assumes his lunches will continue to cost about the same for the next three years.
Manny assumes he will continue to drive about the same number of miles each week over this three year period.
Manny assumes he will still like spinach at the end of three years no matter which choice he makes.
Step 3: Apply Quantitative Tools
Use computational and algebraic tools to quantify the total costs (gas, maintenance/repairs, purchase price) for each scenario over the three years.
Round your answers to the nearest dollar.
Scenario
Total Cost for Three Years
Keep the old car
$
Number
Buy the fuel-efficient used car
$
Number
Step 4: Make an Informed Decision
Based on the information presented what do think Manny should decide?
Option #1: Keep the old car
Option #2: Buy the fuel-efficient used car
Option #
1
Step 5: Evaluate Your Reasoning
Manny plans to start looking at cars and will evaluate his reasoning before he makes a purchase.
( need help on the assignment)

Answers

Manny's graduation date from elementary school and Manny's wages should be removed from his list of variables because they do not apply to the current situation…:

- x²= +x +12=0 .....................................

Answers

Therefore, the solutions of the equation -x² + x + 12 = 0 are x = -3 and x = 6.

What is equation?

In mathematics, an equation is a statement that shows the equality between two expressions, typically separated by an equal sign (=). An equation can contain one or more variables, which are symbols that represent unknown or varying values. The value of the variable(s) can be found by solving the equation.

Here,

The given equation is:

-x² + x + 12 = 0

To solve for x, we can use the quadratic formula:

x = (-b ± √(b² - 4ac)) / 2a

where a, b, and c are the coefficients of the quadratic equation ax² + bx + c = 0.

In this case, we have:

a = -1, b = 1, and c = 12

Substituting these values into the quadratic formula, we get:

x= (-1 ± √(1² - 4(-1)(12))) / 2(-1)

x = (-1 ± √(1 + 48)) / (-2)

x = (-1 ± √(49)) / (-2)

x = (-1 ± 7) / (-2)

There are two solutions:

x = (-1 + 7) / (-2)

= -3

x = (-1 - 7) / (-2)

= 6

To know more about equation,

https://brainly.com/question/28243079

#SPJ1

Select the correct answer.
Which set of coordinates satisfies the equations x − 2y = -1 and 2x + 3y = 12?

A.
(1, 2)
B.
(3, 1)
C.
(3, 2)
D.
(-3, -2)
E.
(3, -2)

Answers

The set of coordinates that satisfy the equation are as follows:

(2, 3)

How to solve equation?

The equation given is as follows:

x - 2y = -1

2x + 3y = 12

Therefore, let's find the set of coordinates that satisfies the equation. Hence,

x - 2y = -1

2x + 3y = 12

multiply equation(i) by 2

Hence,

2x - 4y = -2

2x + 3y = 12

Therefore, subtract the equations

7y = 14

divide both sides of the equation by 7

y = 14 / 7

y = 2

Let's find x as follows

x - 2y = -1

x = -1 + 2(2)

x = -1 + 4

x = 3

Therefore, the solution is (2, 3)

learn more on system of equation here: brainly.com/question/13737751

#SPJ1

If you look at these six numbers, you can see they range from 98 all the way to 642. Would the best interval to use for this group be 10 or 100?

Answers

If you look at the six numbers (101, 203, 407, 512, 98, 642) you can see they range from 98 all the way to 642. the best interval to use for this group be 100.

What is the range?

The right interval to utilize for this set of numbers depends on the aim of the analysis.

If we want to look on the single values and small differences between them, then using interval of 10 would be more better. This would give u room to see small changes that exist between adjacent numbers.

So,  in the event that we need to look on the full range and distribution of the numbers, an interval of 100 would be more better. This would permit us to see how the numbers are shared over a larger area of values.

Learn more about range from

https://brainly.com/question/26098895

#SPJ1

See full question below

If you look at these six numbers, you can see they range from 98 all the way to 642. Would the best interval to use for this group be 10 or 100?

101, 203, 407, 512, 98, 642

Can someone help me in here please

Answers

The addition of the two matrix is [34    -12    -36    -18     58]

                                                        [-42   -8      16      30     8 ]

                                                        [ 24    34     4      34     23]

                                                        [-12    -32    -12    -42   -52]

What is the matrix addition?

The addition of the matrix is calculated as follows;

The result to 4 multiplied by G is calculated as;

4[G] = [32   -20    -32    -8     40]

         [-24   -28     4       36     8 ]

         [ 16     24      12     28     20]

         [-16    -12       0     -40    -36]

2[F] = [2      8    -4    -10     18]

         [-18   20    12   -6      0 ]

         [ 8     10    -8     6      3 ]

         [4     -20  -12    -2    -16]

The addition of the two matrix is calculated as follows;

4[G] + 2[F] =  [34    -12    -36    -18     58]

                     [-42   -8      16      30     8 ]

                     [ 24    34     4      34     23]

                     [-12    -32    -12    -42   -52]

Learn more about matrix here: https://brainly.com/question/2456804

#SPJ1

I need help pleaseeee

Answers

(a) The value of the row operation of -2(row1) + row (2) is [-10     4    -12].

(b) The interchange of row 1 and row 2 is  [0     6    -3]

                                                                       [5    -2     6]

What is the row operation of the matrix?

The value of the row operation of -2(row1) + row (2) is calculated as follows;

-2 (row 1) = 2 [5   -2    6]

-2 (row 1) = [-10     4    -12]

The addition of -2(row1) to row (2) is calculated as follows;

[-10     4    -12] + [0    6    -3]

= [-10   10  - 15]

The interchange of row 1 and row 2 is calculated as follows;

R₁ ↔ R₂

= [5    -2     6]    =   [0     6    -3]

  [0     6    -3]         [5    -2     6]

Learn more about row operations here: https://brainly.com/question/17490035

#SPJ1

PLEASE HELP (1 point)
Let X be a continuous random variable with probability
distribution represented by the graph below.
Find P (X≤2).
f(x)

Answers

The probability P (X≤2). from the probability distribution is 1/3

Evaluating the probability from the probability distribution

A continuous random variable is a type of random variable that can take on any value within a specific range, often an infinite range.

It has a probability distribution function that describes the probability of the variable taking on certain values within that range.

This means that

P (X≤2) =  the f(x) value

From the graph, we have

P (X≤2) = 1/3

Hence, teh value is 1/3

Read more about probability  at

https://brainly.com/question/251701

#SPJ1

The bells obtain a 30-year, $90,000 conventional mortgage at a 10.0% rate on a house selling for $120,000. Their monthly mortgage payment, including principal and interest, is $783.00. They also pay 2 points at closing.
A)What is total amount the Bells will pay for their house.
B) How much of the cost will be interest (including the 2 points)?
C) How much of the first payment on the mortgage is applied to the principal?

Answers

The total amount that the Bells will pay for their house would be $313, 680.

The cost that would be interest is $193, 680

The amount of the first payment going to mortgages is $165. 24

How to find the total amount paid ?

Find the down payment :

Down payment = House price - Mortgage amount = $120,000 - $90,000 = $30,000

Then the points paid at closing :

= $ 90,000 x 0. 02 = $1,800

Total amount paid = Total mortgage payments + Down payment + Points cost

= 281, 880 + 30, 000 + 1, 800

= $ 313, 680

The total that is interest is:

Total interest paid = Total interest paid through monthly payments + Points cost

Total interest paid = $ 191, 880 + $ 1, 800 = $193, 680

The amount of the first payment going to principal is:

Principal portion of first payment = 90, 000 - 89, 834.76 = $165. 24

Find out more on principal at https://brainly.com/question/30163719

#SPJ1

Can someone help me please

here is the picture

Answers

The result of adding -4 (row 1) to row 3 is determined as (0  - 9   2)|12.

What is the result of the row multiplication?

The resultant of the row multiplication in the Matrice is calculated by applying the following method;

row 1 in the given matrices = [1  2  1] | -5

To multiply row by -4, we will multiply each entity by 4 as shown below;

= -4(1 2  1) | -5

= (-4  -8  - 4) |20

To add the result to 3;

(-4  -8  - 4)|20 + (4  - 1   6) | -8

= (0  - 9   2)|12

Thus, the result of the row multiplication is determined by multiplying each entry in row 1, by - 4.

Learn more about row multiplication here: https://brainly.com/question/31531585

#SPJ1

A major credit card company is interested in whether there is a linear relationship between its internal rating of a customer’s credit risk and that of an independent rating agency. The company collected a random sample of 200 customers and used the data to test the claim that there is a linear relationship. The following hypotheses were used to test the claim.

H0:β1=0Ha:β1≠0


The test yielded a t
t
-value of 3.34 with a corresponding p
p
-value of 0.001. Which of the following is the correct interpretation of the p
p
-value?

Answers

The correct interpretation of the p-value is 3.34.

We are given that;

t-value=3.34

Number of customers= 200

Now,

The p-value is the probability of obtaining a test statistic as extreme or more extreme than the observed one, assuming that the null hypothesis is true. In this case, the p-value of 0.001 means that there is a 0.1% chance of getting a t-value of 3.34 or higher (or -3.34 or lower) if the true slope of the linear relationship between the two ratings is zero. The p-value is very small, which indicates that the data provide strong evidence against the null hypothesis and in favor of the alternative hypothesis.

The p-value of 0.001 means that there is a 0.1% chance of getting a t-value of 3.34 or higher (or -3.34 or lower) if there is no linear relationship between the two ratings.

Therefore, by probability the answer will be 3.34

Learn more about probability here;

https://brainly.com/question/9326835

#SPJ1

Help please

Condense to a single logarithm is possible

In(6x^9)-In(x^2)

Answers

The logarithm expression can be simplified to:

In(6x^9)-In(x^2) =7·ln(6x)

How to write this as a single logarithm?

There are some logarithm properties we can use here.

log(a) - log(b) = log(a/b)

log(a^n) = n*log(a)

(these obviously also apply to the natural logarithm)

Now let's look at our expression, it says that:

In(6x^9)-In(x^2)

Using the first rule, we can rewrite this as.

In(6x^9)-In(x^2) = ln(6x^9/x^2)

Now solving the quotient in the argument:

ln(6x^9/x^2) = ln(6x^7) = 7·ln(6x)

That is the expresison simplified.

Learn more about logarithms at:

https://brainly.com/question/13473114

#SPJ1

Using the discriminant

Answers

Answer:

n = 20/3

Step-by-step explanation:

Theory about the discriminant, and how solutions are acquired via the quadratic formula

One variable quadratic equations in the form [tex]0=ax^2+bx+c[/tex] have exactly one solution when the discriminant [tex](b^2-4ac)=0[/tex].

This is because for quadratic equations of the original form, the two solutions are given by the quadratic formula:

[tex]x=\dfrac{-b\pm \sqrt{b^2-4ac}}{2a}[/tex]

which is equivalent to [tex]x=\dfrac{-b}{2a}\pm \dfrac{\sqrt{b^2-4ac}}{2a}[/tex].

The [tex]x=\dfrac{-b}{2a}[/tex] is the axis of symmetry, and the [tex]\dfrac{\sqrt{b^2-4ac}}{2a}[/tex] part is how much the solutions deviate from the axis of symmetry.  If the deviation is zero, then the two solutions are both ON the axis of symmetry, and are the same number, giving exactly one solution.

Addressing the given equation

Putting it into standard form

Given the given equation...

[tex](n-4)u^2+6=8u[/tex]

subtract 8u from both sides to obtain an equation that looks like the standard form equal to zero...

[tex](n-4)u^2-8u+6=0[/tex]

The variable here is "u" instead of "x", so the solutions to this equation would be [tex]u=\dfrac{-b\pm \sqrt{b^2-4ac}}{2a}[/tex], where [tex]a=n-4[/tex], [tex]b=-8[/tex], and [tex]c=6[/tex].

Identifying the discriminant

The discriminant then, with values of a, b, and c, substituted, becomes:

[tex]Discriminant=(-8)^2-4(6)(n-4)[/tex]

If we want the discriminant to equal zero (so that there is exactly one solution for "u"), substitute zero on the left side of the equation, and solve for n.

Solving for n

[tex]0=(-8)^2-4(6)(n-4)[/tex]

[tex]0=64-4(6)(n-4)[/tex]

[tex]0=64-24(n-4)[/tex]

Add -24(n-4) to both sides...

[tex]24(n-4)=64[/tex]

Divide both sides by 24...

[tex]n-4=\dfrac{64}{24}[/tex]

Reduce the right hand side...

[tex]n-4=\dfrac{8}{3}[/tex]

Add 4 to both sides...

[tex]n=\dfrac{8}{3}+4[/tex]

Find a common denominator...

[tex]n=\dfrac{8}{3}+\dfrac{12}{3}[/tex]

Find a common denominator...

[tex]n=\dfrac{20}{3}[/tex]

So, if [tex]n=\dfrac{20}{3}[/tex], then the equation [tex](n-4)u^2+6=8u[/tex] has exactly one solution.  Furthermore, this is the only value of "n" for which the equation has exactly one solution, because it is the only value of "n" for which the discriminant is zero.

In 2015, there were roughly 1 x 10° high school football players and 2x 10° professional football players in the United States. About how many times more high school football players were there?​

Answers

The number of times more high school football players were there is 500.

Given that, in 2015, there were roughly 1×10⁶ high school football players and 2×10³ professional football players in the United States.

Here, the number of times more high school football players were there

= 1×10⁶/2×10³

= 1×10³/2

= 1000/2

= 500

Therefore, the number of times more high school football players were there is 500.

To learn more about the unitary method visit:

brainly.com/question/22056199.

#SPJ2

"Your question is incomplete, probably the complete question/missing part is:"

In 2015, there were roughly 1×10⁶ high school football players and 2×10³ professional football players in the United States. About how many times more high school football players were there?​

A student wants to compare the amount of money that two local movie theaters make over a two-week period for the last nightly showing of a particular movie. The following box plots show the data for the amount of money each theater makes over the period. Compare the median of each box plot.

Answers

Answer:

mt1= 995

mt2=975

Step-by-step explanation:

the line inside the box plot shows where the median is.

Select the reason that best supports Statement 5 in the given proof.

Answers

Where is the given proof

Can yall help me with this pls

Answers

Answer:

x=6.4

Step-by-step explanation:

2/3.2 = 4/x

2x=4(3.2)

2x=12.8

x=6.4

Assertion: 7V5, V2+21 are the irrational number Reason: every integer is a rational number​

Answers

Assertion: Yes, √2 is an irrational number.

Reason: The decimal expansion of √2 is 1.41421356237 which is a non-recurring and non-terminating number.

How to solve

Both (A) and (R) are true and Reason (R) is the correct explanation of Assertion (A).

Assertion: Yes, √2 is an irrational number.

Reason: The decimal expansion of √2 is 1.41421356237 which is a non-recurring and non-terminating number.

All real numbers that are not rational numbers are referred to as irrational numbers in mathematics. In other words, it is impossible to describe an irrational number as the ratio of two integers.

Read more about irrational numbers here:

https://brainly.com/question/20400557

#SPJ1

The Complete Question

Assertion (A): V2 is an irrational number. Reason (R) : Decimal expansion of an irrational number is non-recurring and non terminating???? a) Both (A) and (R) are true and Reason (R) is the correct explanation of Assertion (A) b) Both (A) and ( R) are true but Reason (R) is not a correct explanation of (A) c) Assertion (A) is true and Reason (R) is false d) Assertion (A) is false and Reason (R) is true​

fill it in The value of a certain investment over time is given in the table below. Answer the questions below to determine what kind of function would best fit the data, linear or exponential.
Number of Years Since Investment Made, x
1
2
3
4
Value of Investment ($), f(x)
14,944.54
16,357.04
17,974.72
19,891.18


function would best fit the data because as x increases, the y values change
. The
of this function is approximately
.

Answers

An exponential function would be more appropriate than a linear function, as exponential growth

the best function for the data

To determine which type of function best fits the data, we can analyze the difference in the y values (Value of Investment) as x increases.

Differences between consecutive y values:

16,357.04 - 14,944.54 = 1,412.50

17,974.72 - 16,357.04 = 1,617.68

19,891.18 - 17,974.72 = 1,916.46

The differences between consecutive y values are increasing as x increases. This suggests that an exponential function would be more appropriate than a linear function.

Read more on exponential function here https://brainly.com/question/2456547

#SPJ1

Which of the points plotted is farther away from (−7, 8), and what is the distance?

A: Point (5, 8), and it is 11 units away
B: Point (5, 8), and it is 13 units away
C: Point (−7, −5), and it is 12 units away
D: Point (−7, −5), and it is 13 units away

Answers

The distance between point (−7, 8) and point (5, 8) is 12 units (since they are on the same horizontal line). The distance between point (−7, 8) and point (−7, −5) is 13 units (using the Pythagorean theorem). Therefore, the point that is farther away is option D: Point (−7, −5), and it is 13 units away.

Let S be a collection of subset of {2000, 2001, 2002, …, 2020} such that intersection of any two sets in S is nonempty. What is the maximum cardinality of S?

Answers

The maximum cardinality of S is 20.

How to find the maximum cardinality ?

To increase the number of subsets within S, our goal must be to find as many non-empty subset intersections as possible. This is achieved by constructing subsets that possess only one shared element. In this particular case, any two sets contained in the collection will include the element 2000 in their intersection causing it to become a valid subset.

Now, we shall investigate whether an additional subset can be added to S. If an extension exists without including 2000, its intersection with the previous 20 subsets would negligibly have no common elements and would hence not be regarded as a valid subset for addition into S.

As a result, a total of 20 subsets represent maximum cardinality for set S per these conditions.

Find out more on maximum cardinality at https://brainly.com/question/30701593

#SPJ1

The expression 5(2) gives the number of leaves on a plant as a function
of the number of weeks since it was planted.
What does 2 represent in this expression?
Choose 1 answer:
B
The plant was planted 2 weeks ago.
There were initially 2 leaves on the plant.
The number of leaves is multiplied by 2 each week.

Answers

Question:The expression 5(2) gives the number of leaves on a plant as a functionAnswer:

C: the number of leaves is multiplied by 2 each week

Step-by-step explanation

The proper expression ought to be 5(2) ^ t =5(2)^{t}

where t speak to the number weeks after it planted.

On the off chance that we put to the time(t) we are going know how numerous clears out at first are. The calculation will be:

5(2)^{t}= 5(2)^{0} =5

This appear that coefficient 5 decide how much the takes off at first are.

Let see how the number going for distinctive esteem of t. On the off chance that its 1 week after plant, the leaf will ended up:

5(2)^{t}= 5(2)^{1} = 10

At that point the number of takes off 2 weeks after planted will be:

5(2)^{t}= 5(2)^{2}= 20

The number keeps twofold each week since coefficient 2 speaks to the proportion of the development of the work and the proportion decides how much takes off duplicated each week.

So, the reply will be C.

if a clock shows it is 3 o'clock, how could you describe the smaller angle made my the two hands of the clock? solve this problem any way you choose

Answers

The smaller angle made by the two hands of the clock at 3 o'clock is 90 degrees.

What is an angle?

A figure known as an angle is created by two rays or line segments that meet at a place known as the vertex of the angle. The sides or legs of the angle are other names for the rays or line segments.

According to question:

To determine the smaller angle made by the two hands of a clock when it is 3 o'clock, we can use the following formula:

angle = |(11/2) * m - 30h|

where:

m is the number of minutes past the hour (in this case, since it is 3 o'clock, m = 0)

h is the hour (in this case, h = 3)

By using this formula, we get:

angle = |(11/2) * 0 - 30(3)| = |0 - 90| = 90

Therefore, the smaller angle made by the two hands of the clock at 3 o'clock is 90 degrees.

To know more about angle visit:

https://brainly.com/question/28451077

#SPJ1

Researchers are analyzing the total annual cost for the top ranked universities. They need to display the data in a way that is helpful
for relaying information regarding the data.

Identify the advantage of using a dotplot (choose one) :

A- Easily identify outliers

B- Easily determine the total number of data points for larger data sets

C- Cannot determine the minimum or maximum

D-Cannot identify the exact median

Answers

One advantage of using a dot plot is that it makes it easy to identify outliers. Option A is correct.

An outlier is a data point that is significantly different from the others in the data set, either in terms of its value or its position within the range of values. In a dot plot, outliers are represented by individual dots that are far away from the other dots, either above or below them.

By looking at the dot plot, researchers can quickly identify any outliers and investigate them further to determine why they are different and whether they should be included or excluded from the analysis.

Hence, A. is the correct option.

To know more about dot plot here

https://brainly.com/question/21862696

#SPJ1

Find the slope of the line that passes through the given points.
​(−1​,−1​) and ​(3​,−4​)

Answers

Answer: [tex]\frac{-3}{4}[/tex]

Step-by-step explanation:

   To find the slope of the line, we will use change in y over change in x.

[tex]\displaystyle \frac{y_{2} -y_{1} }{x_{2} -x_{1} } =\frac{-4--1}{3--1} =\frac{-4+1}{3+1} =\frac{-3}{4}[/tex]

Other Questions
Wholesale price: $17 retail price: $25 markup on retail: ?a. 8%b. 32%c. 47%d. 14% NEED HELP FAST!!!! Please answer both questions In chapter 6 of the Great Gatsby, what is the implication surrounding Dan Codys death?No plagiarizing! Which nation began rearming around the same time that radio became a common technology? a. Hitlers Germanyb. Churchill's Britainc. De Gaulle's Granced. Victor Emmanuel's Italy Find the following. f'(2) if f(x) = -8x^-1 + 5x$-2 O 13/14O -3/4O -13/4O The mean of six values is 7. There is one outlier thatpulls the mean higher than the center. What could thedata set be? What is the mean without the outlier? Write a letter to the Mayor of your municipality or the Chairperson of your rural municipality requesting him/ her to involve the representatives of children in the decision making process related to children's issues in your municipality / rural municipality if you were to design a vaccine to prevent infection by a mucosal pathogen, such as vibrio cholerae, which antibody class would be most effective in preventing colonization in the intestinal tract? iga ige igd igm igg Why is specialization a common result of free trade?A. Worker safety agreements encourage production that uses skilledlabor.B. Competition encourages countries to specialize in what they dobest.C. Trade agreements determine what each country must produce.D. Loss of import tax revenues forces countries to limit what theymanufacture. 28 Laney's art teacher, Mr. Brooks, has four different colors of clay. Laney and some of her classmates will be using this clay to make different figures. The following table shows the number of pounds of each color of clay Mr. Brooks has available. Clay Amount Color (pounds) Biue 11 5 Green 8 Yellow 2 Red 15 4. Use this information to help you answer parts A through E of this problem. Part A Laney noticed that one color of clay was exactly twice the amount of clay of another color. Which color of clay weighs exactly twice the number of pounds of another color of clay? A. Blue B. Green C. Yellow D. Red. The System Is Only As Good As Its Individual Parts Introduction As more individuals stress a holistic or systems thinking approach to fixing healthcare organizations and the US healthcare systems, we are seeing the influence, both positive and negative, one department or process may have on an entire organization Describe the imponce of understanding the tools applicable to improvements at the department or process level relative to systems thinking. which one of the following is the correct order of the electromagnetic spectrum from low to high frequencies? select one: a. radio waves, uv, x-rays, microwaves, infrared, visible, gamma rays b. radio waves, infrared, microwaves, uv, visible, x-rays, gamma rays c. radio waves, microwaves, infrared, visible, uv, x-rays, gamma rays d. radio waves, infrared, x-rays, microwaves, uv, visible, gamma rays If the genotype for a trait is hybrid, the trait expressed will be a reccessiveb dominantc purebredd honozygous It is April, and Hans Anderson is planting his barley crop near Plunkett, Saskatchewan. He is concerned about losing his farm if his operations result in a loss at the end of the season. He expects to harvest 3,000 tonnes of barley and sell it in October. Futures contracts are available for October delivery with a futures price of $200 per tonne. Options with strike price of $200 per tonne are also available; puts cost $15 and calls cost $20. a) If Hans takes a short position in futures on 3,000 tonnes of barley for delivery in October, what will be the total net amount received by Hans (for all 3,000 tonnes) if the price of barley in October is as follows: i) $150 ii) $200 iii) $250b) If Hans can purchase put options on 3,000 tonnes of barley, what will be the total net amount received by Hans (for all 3,000 tonnes) if the price of barley in October is as follows: i) $150 ii) $200 iii) $250 In the equationIn the equationT = -mv,T = = my, find the value of T when m = 50 and v= 2hon simplify. Finish breaking the sentence into three separate ideas.His prosthetic leg, made with metal and other manufacturedmaterials, doesn't look like a biological limb, but it works aswell as one-and sometimes even better.The climber's prosthetic legThe prosthetic leg doesn't look likeThe prosthetic legmetal anda biological limb. the radicalism of the sa worried the more conservative groups that had helped make hitler chancellor, so in a bid to curtail the sa on the night of june 30, 1934, more than a thousand high-ranking sa officials, including several of hitler's oldest associates, were executed in a bloody purge known as What naturally occurring phenomenon does geothermal power takes advantage of to produce electricity? (WILL GIVE BRAINLIEST TO THE CORRECT ANSWER)solar energyenergy from organic matterunderground heatenergy from moving water A researcher would like to examine how the chemical tryptophan, contained in foods such as turkey, can reduce mental alertness. a sample of n = 9 college students is obtained, and each students performance on a familiar video game is measured before and after eating a traditional thanksgiving dinner including roasted turkey. the average mental alertness score dropped by md= 14 points after the meal with ss= 1152 for the difference scores.a. is there is significant reduction in mental alertness after consuming tryptophan versus before? use a one-tailed test with = .05.b. compute r2 to measure the size of the effect. SEP Plan and Carry Out Investigations Suppose that you were a geologist trying to figure out how a long and narrow sea, such as the Red Sea, formed. What geologic features would you look for to determine whether the current shape of the sea is a result of seafloor spreading or ocean subduction? a